Matrices Question: Solving a System with No Solution for k | Help Needed

  • Thread starter danizh
  • Start date
  • Tags
    Matrices
In summary, the system of equations represented by the reduced augmented matrix will have no solution for values of k equal to 0, 1, or any value greater than 2. This is because for these values, the z-coefficient becomes 0 while the constant is not 0, making the system unsolvable. However, for some other values of k, such as -1, there may be a solution depending on the other coefficients in the matrix.
  • #1
danizh
16
0
Please help me solve this problem associated with matrices; I can't seem to figure it out.

The following reduced augmented matrix represents a system of equations of three plans:

1 0 0 | 3
0 1 0 | 4
0 0 (k^2)-k | k

For what value(s) of k will this system have no solution?

:eek:
 
Last edited:
Physics news on Phys.org
  • #2
If the z-coëfficiënt becomes 0 while the constant isn't 0, your system won't have a solution.

So [tex]k^2 - k = 0 \Leftrightarrow k = 0 \vee k = 1[/tex].

But for k = 0, the constant is 0 as well so the last equation is no longer independant, then you have a 2x3 system (which has infinite solutions).

For k = 1, it has no solution :smile:
 
Last edited:
  • #3
Awesome, thanks for the help! :smile:
Wouldn't "+1" have no solution as well?
 
  • #4
I mean +1 lol, it has a solution for -1 :wink:

I'll correct.

---

Done.

Since, for k = -1, the last row becomes: 0 0 2 | 1 => 2z = 1 => z = 1/2
 
  • #5
But (-1)^2 - 1 = 0 as well. So there are no solutions for "+/- 1," I suppose.
Please correct me if I am wrong. :uhh:
 
  • #6
Watch out, mind your minus! For k = -1, you get:

[tex]k^2 - k \Rightarrow \left( { - 1} \right)^2 - \left( { - 1} \right) = 1 + 1 = 2[/tex]

Which is what I wrote at the end of my previous post.
 
  • #7
Sorry, I missed that. :tongue2:
Thanks for the help!
 
  • #8
No problem :smile:
 
  • #9
I don't think that's true, since anything greater than 2 seems to have no solution as well.
 
  • #10
danizh said:
I don't think that's true, since anything greater than 2 seems to have no solution as well.
How is that?

Take k = 3, the last row becomes: 0 0 6 | 3 => 6z = 3 <=> z = 1/2
 

1. What is a matrix?

A matrix is a rectangular array of numbers or variables arranged in rows and columns. It is often used to represent a system of equations or to perform calculations in linear algebra.

2. How do you solve a system of equations using matrices?

To solve a system of equations using matrices, you need to first create an augmented matrix by combining the coefficients and constants of the equations. Then, you can use row operations to reduce the matrix to row-echelon form or reduced row-echelon form, which will give you the solutions to the system.

3. What does it mean when a system has no solution for a particular variable?

If a system has no solution for a particular variable, it means that the value of that variable does not satisfy all of the equations in the system. In other words, there is no single value that can make all of the equations in the system true simultaneously.

4. Can a system have no solution for all variables?

Yes, it is possible for a system to have no solutions for all variables. This occurs when the equations in the system are inconsistent, meaning that there is no combination of values that can satisfy all of the equations at the same time.

5. How do you know if a system has no solution for a particular variable?

You can determine if a system has no solution for a particular variable by using row operations to reduce the augmented matrix to row-echelon form or reduced row-echelon form. If you end up with a row of zeros and a non-zero constant in the corresponding column, then there is no solution for that variable.

Similar threads

  • Introductory Physics Homework Help
Replies
1
Views
1K
  • Introductory Physics Homework Help
Replies
10
Views
899
  • Introductory Physics Homework Help
2
Replies
40
Views
2K
  • Introductory Physics Homework Help
Replies
6
Views
231
  • Introductory Physics Homework Help
Replies
17
Views
374
Replies
1
Views
141
  • Introductory Physics Homework Help
Replies
29
Views
913
  • Linear and Abstract Algebra
Replies
8
Views
784
  • Linear and Abstract Algebra
Replies
2
Views
2K
  • Introductory Physics Homework Help
Replies
2
Views
1K
Back
Top